You are on page 1of 17

Quant Updater – 20 | 24 June

Daily Quant Free Magazine for Bank Exam Students

24 June 2023 | SET - 20

Any Error or Doubt Found Please Mail us to :-


WhatsApp @ 8445784192
1
Quant Updater – 20 | 24 June
Daily Quant Free Magazine for Bank Exam Students

“जिसने संघर्ष को अपना साथी बना जिया


उसने ही इस दुजनया में
खुद को काबबि बना जिया.!!

मेरा नाम अरुण जसिं ह रावत हैं मैं साथियों जो BANK और Insurance Exams की परीक्षा देते है उन्हें Quant विषय
समझाने में मदत करता हूँ
यूँहा आज मैं आपसे एक मेंटर की तरह नहीं बल्कि एक दोस्त की तरह बात कर रहा हूँ !
मेरी Journey भी आपकी ही तरह ही है की हर पररिार का सपना उनका बेटा या बेटी एक सरकारी नौकरी करे
तो MATHS के डर ने काफी लोग को इस रेस में हमेशा पीछे ही रखा है इस साल मैंने कोशशश की है हर बच्चे को
बैंक की तैयारी के थलए बैंक एग्जाम की फीस के बराबर ही कोसस फीस देनी पड़े ले वकन क्लास के बाद प्रैल्किस
करने के थलए Quality Content जब बच्चो को थमल नहीं पा रहा िा तो तो सोचा की इस पर भी काम करते है
और सारे बच्चों के थलए एक फ्री मैगज़ीन तैयार की है शजसका नाम है " QUANT UPDATER "
दोस्तों आज भी अपने भारत में ऐसे बहुत सारे बच्चे है जो कोई कोसस या बैच खरीद नहीं पाते है अरुण सर ने उनके
थलए मात्र 199 रुपए में अपना एक कोसस लॉन्च वकया है आप उस कोसस की मदद से अपना फाउं डेशन अपना बेस
तैयार कर सकते है | दोस्तों बहुत अच्छा लगता है जब कोई स्टूडेंट्स मैसेज करता है वक सर आपकी गाइडन्स की
िजह से मेरा इग्ज़ैम हो गया है या फफर मैंने मैथ्स में 35 में 35 स्कोर वकया है ये जानकर अच्छा लगता है की मेरी
की हुई मेहनत का फल लोगों को थमल रहा है और उसकी सक्सेस का एक छोटा सा रहस्सा मैं भी हूँ !
इसी सोच को नया आयाम देने के थलए मैं ले कर आया हु जो की एक मैगजीन है " Quant Updater" शजसमे
आपको रोजाना सभी Quant Topic जैसे Simplification, Approximation, Number series, Data
interpretation aur Arithmetic के question प्रैल्किस करने के थलए थमले गा | इसके अलािा आपको
Detailed Solution फदया गया है जो आपकी प्रॉब्ले म को और आसान कर देगा |

Dosto is magazine ki help se aap apni speed improve kar sakte ho aur apne quant ke dar
ko hara sakte ho. ab bhi koi result aaye aap apne sapne ko pura hota dekhe.

अगर आपको कोई DOUBT रहता है वकसी सिाल पे तो आप मुझे मेल कर सकते है
Mathsbyarunsir007@gmail.com

दोस्तों आप सभी लोगों को एक नौकरी थमल जाए बस इसी सपने के साि आप मेहनत करो मैं हमेशा आपके
साि हूँ | दोस्तों शसले क्शन के बाद इं तजार रहेगा आपके एक मैसेज की सर मैंने कर फदखाया ! और आज मैं ने
अपने माूँ बाप और आपका नाम रोशन कर फदया है

Reasoning, English GA आप PRACTICE कर िो | Maths जसर्ष अरुण सर को कर िो | |

आपका दोस्त | आपका मेंटर


अरुण शसिं ह राित
( Maths by Arun Sir )
8445784192

Any Error or Doubt Found Please Mail us to :-


WhatsApp @ 8445784192
2
Quant Updater – 20 | 24 June
Daily Quant Free Magazine for Bank Exam Students

CONTENT
INDEX

Chapter – 1 | Speed Math - 4 Questions – 4 Minutes

1. Question Paper …………………………….. 4-5

2. Solution ……………………. 12

Chapter – 2 | Data Interpretations Graph Based - 3 Questions – 6 Minutes

1. Question Paper …………………………….. 6-7

2. Solution ……………………. 12-13

Chapter – 3 | Caselet Data Interpretation - 6 Questions – 12 Minutes

1. Question Paper …………………………….. 8-9

2. Solution ……………………. 13-16

Chapter – 4 | Arithmetic Based Questions - 4 Questions – 8 Minutes

1. Question Paper …………………………….. 10-11

2. Solution ……………………. 16-17

Any Error or Doubt Found Please Mail us to :-


WhatsApp @ 8445784192
3
Quant Updater – 20 | 24 June
Daily Quant Free Magazine for Bank Exam Students

Chapter – 1 | Speed Math

Directions: In the following approximation questions, three statements are given. Find which of them is/are true.
थनम्नथलखखत सल्किकटन प्रश्नों में, तीन किन फदए गए हैं। ज्ञात कीशजये वक उनमें से कौन सा/से सत्य है/हैं।
1. 55.08% of 149.98 + √(28.08 × 6.95) − 11 1/9% 𝑜𝑓 998.99 = x
I. Value of x lies between -15 to -13.
II. x is a negative fraction number greater than -14.
III. x is a multiple of 5.
I. x का मान -15 से -13 के बीच है।
II. x -14 से अधिक एक ऋणात्मक थभि संख्या है।
III. x, 5 का एक गुणज है.
(a) Only I (b) Both I and II (c) Both I and III (d) All the three (e) None of the three

322.98
2. 149.99% of 20 + + √x =(8.99)²
17.10
I. x is a multiple of both 9 and 11
II. x is a square of a natural number lying between 31 and 34.
III. x is an odd number and multiple of 99.
I. x 9 और 11 दोनों का एक गुणज है
II. x 31 और 34 के बीच स्थित एक प्राकृवतक संख्या का एक िगस है।
III. x एक विषम संख्या है और 99 का एक गुणज है।
(a) Only I (b) Both I and II (c) Both I and III (d) All the three (e) None of the three

Direction (3-4) : Given below are three quantities named 1, 2 and 3. Based on the given information, you have to
determine the relation among the three quantities. You should use the given data and knowledge of Mathematics
to choose between the possible answer. The options represent the relations among three quantities.
नीचे तीन मात्राएूँ 1, 2 और 3 दी गयी है। दी गई जानकारी के आिार पर, आपको तीन राशशयों के बीच संबंि थनिासररत करना होगा। आपको
संभावित उत्तर के बीच चयन करने के थलए गशणत के फदए गए डेटा और ज्ञान का उपयोग करना चारहए। विकल्प तीन मात्राओ ं के बीच संबंिों
का प्रवतथनधित्व करते हैं।
A. < B. > C. ≤ D. ≥ E. =

3. Quantity 1: Find the next term./ अगला पद ज्ञात कीशजए।


1, 3, 5, 31, ?
Quantity 2: Find out the wrong number./ गलत नंबर का पता लगाएं ।
11, 15, 31, 67, 132, 231.
Quantity 3: Find the next term. / अगला पद ज्ञात कीशजये
28, 32, 41, 66, ?
(a) A B (b) B B (c) A A (d) D A (e) None of these

Any Error or Doubt Found Please Mail us to :-


WhatsApp @ 8445784192
4
Quant Updater – 20 | 24 June
Daily Quant Free Magazine for Bank Exam Students

4. Quantity 1: Find the next term. / अगला पद ज्ञात कीशजए।


5, 10, 40, 80, (?)
Quantity 2: Find out the wrong number. / गलत नंबर का पता लगाएं
15, 31, 63, 127, 255, 510
Quantity 3: Find the next term. / अगला पद ज्ञात कीशजए।
342, 353, 341, 354, (?)
(a) A B (b) B C (c) C A (d) D A (e) None of these.

Any Error or Doubt Found Please Mail us to :-


WhatsApp @ 8445784192
5
Quant Updater – 20 | 24 June
Daily Quant Free Magazine for Bank Exam Students

Chapter – 2 | Data Interpretations Graph Based


Graph Based
Directions (5-7): The following table shows the number of days for which 4 individuals Prateek, Suleman, Chandan
and Vikas worked on 5 different assignments numbered A to E. It also shows the part of respective assignment
that could not be completed by individuals in time.
Some values are missing which are denoted by symbol (-). With the help of information in questions and table
below answer the questions that follow.
फदशा-थनदेश: थनम्न ताथलका उन फदनों की संख्या को दशासती है शजनके थलए 4 व्यधि प्रतीक, सुलेमान, चंदन और विकास ने 5 अलग-अलग
असाइनमेंट A से E पर काम वकया। यह संबंधित असाइनमेंट का िह भाग भी फदखाता है जो समय में व्यधियों द्वारा पूरा नहीं वकया जा सका।
कुछ मान गायब हैं जो प्रतीक (-) द्वारा थनरूवपत हैं। प्रश्नों में जानकारी की मदद से और नीचे दी गई ताथलका में उन प्रश्नों का उत्तर दें जो
अनुसरण करते हैं।
Assignment Number of days for individuals worked on different Part of assignment remain
Number assignments uncompleted after work
Prateek Suleman Chandan Vikas
A 6 6 2 3 1
3
B - 3 4 - -
C - - - - -
D 5 3 2 - 1
6
E 2 2 3 5 1
12

5. Prateek and Suleman started working on Assignment A. They completed 1/3rd of work after which they left and
Chandan joined the assignment. Chandan can complete the whole assignment in 12 days. After Chandan
worked for his assigned number of days, Vikas joined the assignment and worked for his assigned number of
days. Find the number of days in which Vikas can complete the whole Assignment A?
Prateek और Suleman ने असाइनमेंट A पर काम करना शुरू कर फदया। उन्होंने 1/3 िां काम पूरा वकया, शजसके बाद िे चले गए और
चंदन असाइनमेंट में शाथमल हो गए। चंदन 12 फदनों में पूरे असाइनमेंट को पूरा कर सकते हैं। चंदन द्वारा फदए गए फदनों की संख्या के थलए
काम करने के बाद, विकास ने असाइनमेंट पर काम करना शुरू कर फदया और अपने थनिासररत फदनों के थलए काम वकया। उन फदनों की
संख्या का पता लगाएं शजनमें विकास पूरे असाइनमेंट को पूरा कर सकते हैं ?
(a) 15 days (b) 20 days (c) 18 days (d) 25 days (e) 12 days

6. Prateek who can complete assignment B in 20 days, worked for 6 days. The ratio of number of days in which
Suleman and Chandan can complete Assignment B alone is 5 : 8. Vikas could not come to work for assignment
13
B. Find the number of days in which Suleman and Chandan can complete of assignment B together, given
30
that if Vikas who can complete work in 12 days had also joined the assignment for 4 days, the assignment would
have been completed.

Any Error or Doubt Found Please Mail us to :-


WhatsApp @ 8445784192
6
Quant Updater – 20 | 24 June
Daily Quant Free Magazine for Bank Exam Students

Prateek जो 20 फदनों में असाइनमेंट B को पूरा कर सकता है, 6 फदनों के थलए काम वकया। उन फदनों की संख्या का अनुपात शजसमें
सुलेमान और चंदन अकेले असाइनमेंट बी को पूरा कर सकते, 5: 8 हैं. विकास असाइनमेंट बी के थलए काम करने नहीं आ सका। उन फदनों
13
की संख्या का पता लगाएं शजनमें सुलेमान और चंदन असाइनमेंट B के को एक साि पूरा कर सकते हैं। यह देखते हुए वक अगर विकास
30
जो 12 फदनों में काम पूरा कर सकता है, अगर िह भी 4 फदनों के थलए असाइनमेंट में शाथमल हो गया होता, तो असाइनमेंट पूरा हो गया होता।
(a) 3 days (b) 6 days (c) 5 days (d) 4 days (e) 8 days

7. Prateek, Ankit and Vikas worked on assignment E. Suleman and Chandan alone can complete whole
3
assignment E in 20 and 30 days respectively. Ankit who is times efficient than Suleman and Chandan together
2
replaces both of them and worked for same number of days for which Suleman and Chandan had to work.
Prateek completed 1/12th of the work. Find in how many days all Prateek, Suleman, Chandan, and Vikas can
complete the assignment E together.
Prateek, Ankit और Vikas ने असाइनमेंट E पर काम वकया। सुलेमान और चन्दन पुरे असाइनमेंट E को क्रमशः 20 और 30 फदनों में पुअर
3
कर सकते हैं. अंवकत जो सुलेमान और चंदन की तुलना में गुना कुशल है, दोनों को एक साि स्वयं से बदल देता है और समान फदनों के
2
थलए काम करता है शजसके थलए सुलेमान और चंदन को काम करना िा। Prateek ने काम का 1/12 िां रहस्सा पूरा वकया। यह पता करें
वक सभी Prateek, Suleman, Chandan, और Vikas वकतने फदनों में असाइनमेंट E को एक साि पूरा कर सकते हैं।
(a) 3 days (b) 6 days (c) 8 days (d) 11 days (e) 9 days

Any Error or Doubt Found Please Mail us to :-


WhatsApp @ 8445784192
7
Quant Updater – 20 | 24 June
Daily Quant Free Magazine for Bank Exam Students

Chapter – 3 | Caselet Data Interpretation Based

Directions (8-10) : Study the following information carefully and answer the questions given beside.
Cristiano Ronaldo scored goals against different countries in three different years.
NOTE: Total goals scored in a year= Argentina + England + Others
2019: The total goals scored in 2019 were 120. The goals scored against England were 1/3rd of the goals against
Others in 2020. The average goals scored against Argentina and England was 30.
2020: The total goals scored against Argentina and Others was 120. The ratio of the total goals scored against
Others in 2019 to that of the total goals scored against Others in 2020 is 4:3. The total goals scored against England
in 2020 were equal to the total goals scored against England in 2021.
2021: The sum of the total goals scored against Argentina and England is equal to the total goals scored against
Others. The total goals scored in 2021 were 140. The total goals scored against Argentina were twice of the goals
scored against England in 2019.
जनदेश: थनम्नथलखखत जानकारी का साििानीपूिसक अध्ययन करें और बगल में फदए गए प्रश्नों के उत्तर दें।
वक्रस्टस्टयानो रोनाल्डो ने तीन अलग -अलग िषों में विथभि देशों के खखलाफ गोल वकए।
नोट: एक िषस में कुल गोल = (अजेंटीना + इं ग्लैं ड + अन्य) के खखलाफ वकये गए गोल
2019: 2019 में बनाए गए कुल गोल 120 िे। इं ग्लैं ड के खखलाफ वकए गए गोल 2020 में अन्य के खखलाफ वकये गए गोल के 1/3 िे। अजेंटीना
और इं ग्लैं ड के खखलाफ बनाए गए औसत गोल 30 िे।
2020: अजेंटीना और अन्य के खखलाफ वकए गए कुल गोल 120 िे। 2019 में अन्य के खखलाफ बनाए गए कुल गोल का 2020 में अन्य के
खखलाफ वकए गए कुल गोल से अनुपात 4: 3 िा। 2020 में इं ग्लैं ड के खखलाफ वकए गए कुल गोल 2021 में इं ग्लैं ड के खखलाफ वकए गए कुल
गोल के बराबर िे।
2021: अजेंटीना और इं ग्लैं ड के खखलाफ वकए गए कुल गोल का योग अन्य के खखलाफ वकए गए कुल गोल के बराबर िे। 2021 में बनाए गए
कुल गोल 140 िे। अजेंटीना के खखलाफ वकए गए कुल गोल 2019 में इं ग्लैं ड के खखलाफ वकए गए गोलों के दोगुने िे।

8. What were the total goals scored in 2020?


2020 में कुल गोल क्या िे?
(a) 110 (b) 150 (c) 160 (d) 140 (e) 100

9. What is the ratio of the total goals scored against Argentina in 2019 to that of the total goals scored against
England in 2021?
2019 में अजेंटीना के खखलाफ वकए गए कुल गोल का 2021 में इं ग्लैं ड के खखलाफ वकए गए कुल गोल से अनुपात क्या है ?
(a) 7 : 9 (b) 9 : 7 (c) 11 : 7 (d) 9 : 8 (e) 11 : 13

10. What is the difference between the total goals scored against Others in 2019 to the total goals scored against
Others in 2020?
2019 में दूसरों के खखलाफ वकए गए कुल गोल और 2020 में दूसरों के खखलाफ वकए गए कुल गोल के बीच क्या अंतर है?
(a) 15 (b) 20 (c) 10 (d) 30 (e) 25

Any Error or Doubt Found Please Mail us to :-


WhatsApp @ 8445784192
8
Quant Updater – 20 | 24 June
Daily Quant Free Magazine for Bank Exam Students

Directions (11-13) : Read the information given in paragraph carefully and answer the following questions.
Mahesh has a certain amount with him, out of which, 20% he invests in mutual fund, 15% he spent on shopping
2
and 25% on rent and food. Out of remaining amount, he invested 41 ( ) % in scheme A for 2 years which offers
3

20% of annual compound interest and remaining in scheme B for 2 years that offers 20% simple rate of interest.
If total amount of interest received from both the schemes together is Rs.2500 more than the amount spent
on shopping. From the interest he obtained from scheme B he purchased some tables and some chairs. Total
units (both chair and table) purchased by him is 16 and probability of selecting 2 chairs at random is 11/20.
2
From the interest received from scheme A he purchased another item X at 16 ( ) % discount.
3

जनदे श : पररच्छे द में दी गई जानकारी को ध्यानपूिसक परिए और थनम्नथलखखत प्रश्नों के उत्तर दीशजए।
महे श के पास एक थनथित िनराशश िी, शजसमें से िह 20% म्यूचुअल फंड में थनिेश करता है , 15% खरीदारी पर और 25% वकराए और
2
भोजन पर व्यय करता है । शेष िनराशश में से 41 ( ) % िनराशश को 20% िावषि क चक्रिृद्धि ब्याज दर पर 2 िषों के थलए उसने योजना A
3

में थनिेश वकया और शेष िनराशश को 20% सािारण ब्याज दर पर 2 िषों के थलए योजना B में थनिेश वकया। एक साि दोनों योजनाओ ं से
अशजि त ब्याज की कुल िनराशश, खरीदारी पर व्यय की गई िनराशश से रु. 2500 अधिक है । योजना B से अशजि त ब्याज की िनराशश से
उसने कुछ मेज और कुछ कुशसि याूँ खरीदा। उसके द्वारा खरीदी गई कुल यूथनट (कुसी और मेज दोनों) 16 हैं तिा यादृच्छच्छ क रूप से 2 कु शसि यों
11 2
के चयन करने की प्राधयकता है । योजना A से अशजि त ब्याज की िनराशश से उसने एक अन्य िस्तु X को 16 ( ) % की छूट पर खरीदा।
20 3

11. If two items are selected at random among purchased chairs and tables by Mahesh, then what is the
probability that both the items are either tables or one is chair and another is table?
यफद महे श द्वारा खरीदी गई कु शसि यों और मेजों में से यादृच्छच्छ क रूप से दो िस्तुओ ं का चयन वकया जाता है , तो दोनों िस्तुओ ं के या तो मेज
या एक िस्तु के कुसी और दूसरी िस्तु के मेज होने की प्राधयकता वकतनी है ?
1 7 47 9
(a) (b) (c) (d) (e) None of these
6 30 120 20

12. If marked up per cent on item X was 32% and shopkeeper sold the same item to another customer Suresh
at 15% profit, then what is the discount per cent offered by shopkeeper to Suresh on that item?
यफद िस्तु X पर मूल्य-िृद्धित प्रवतशत 32% िा और दुकानदार ने समान िस्तु को दूसरे ग्राहक सुरेश को 15% के लाभ पर बेचा, तो दुकानदार
द्वारा सुरेश को उस िस्तु पर प्रदान की जाने िाली छूट प्रवतशत वकतना है ?
29 2 9 4
(a) 12 ( ) % (b) 15 ( ) % (c) 8 ( ) % (d) 16 ( ) % (e) None of these
33 3 11 9

13. If from the interest received from scheme A, Mahesh started a business with Sunil who invested Rs.9000
and after 4 years from the start of business, Mahesh withdraws Rs.7000 and total profit amount from the
business at the end of 7 years is Rs.21250, then what is the profit amount received from Sunil?
यफद योजना A से अशजि त ब्याज की िनराशश से महे श ने सुनील के साि एक व्यिसाय शुरू वकया और सुनील ने रु.9000 का थनिेश वकया
तिा व्यिसाय शुरू होने के 4 िषों के बाद महे श ने रु.7000 िापस थनकाल थलया। यफद 7 िषों के अंत में व्यिसाय से अशजि त लाभ की कुल
िनराशश रु.21250 है, तो सुनील को प्राप्त लाभ की िनराशश वकतनी है ?
(a) Rs.10500 (b) Rs.11250 (c) Rs.9250 (d) Rs.12750 (e) None of these

Any Error or Doubt Found Please Mail us to :-


WhatsApp @ 8445784192
9
Quant Updater – 20 | 24 June
Daily Quant Free Magazine for Bank Exam Students

Chapter – 4 | Arithmetic Based Questions

14. Which of the following pair of symbols will define the relation between Quantity I and Quantity II and between
Quantity II and Quantity III respectively?

थनम्नथलखखत में से कौन-सा प्रतीक जोड़ा क्रमशः मात्रा I और मात्रा II तिा मात्रा II और मात्रा III के बीच संबंि को पररभावषत करेगा?

A. = B. > C. < D. ≥ E. ≤ F. # (संबंि िावपत नहीं वकया जा सकता)

मात्रा I: दो पासे को एक साि उछालने पर शीषस पर 8 अंक आने की प्राधयकता।

मात्रा II: तीन शसक्कों को एक साि उछालने पर अधिकतम दो heads आने की प्राधयकता।

मात्रा III: दो पासों को एक साि उछालने पर दोनों पासे पर समान अंक आने की प्राधयकता।

A. = B. > C. < D. ≥ E. ≤

F. # (relation cannot be established)

Quantity I: Probability of getting a sum as 8 on the top when two dices are rolled together.

Quantity II: Probability of getting at most two heads when three coins are tossed together.

Quantity III: Probability of getting same number on both the dice when two dices are rolled together.

(a) A, B (b) B, C (c) C, D (d) C, B (e) E, D

15. Which of the following pair of symbols will define the relation between Quantity I and Quantity II and between
Quantity II and Quantity III respectively?

A. = B. > C. < D. ≥ E. ≤

F. # (relation cannot be established)

Quantity I: A train can cross a pole in 8.8 seconds and a 207 metres long platform in 18 seconds. Find the time
taken by the train to cover a distance of 567 km if it increases its speed by 40%.

Quantity II: A boat can travel 160 km in upstream as well as same distance in downstream in 9 hours while the
same boat can travel 240 km in upstream in 7.5 hours. Find the time taken by the boat to cover 180 km in still water.

Quantity III: Find the time taken by a car travelling with a speed of 17.5 m/s to cover a distance of 315 km.

थनम्नथलखखत में से कौन-सा प्रतीक जोड़ा क्रमशः मात्रा I और मात्रा II तिा मात्रा II और मात्रा III के बीच संबंि को पररभावषत करेगा?A. =B.
>C. <D. ≥E. ≤F. # (संबंि िावपत नहीं वकया जा सकता है)

मात्रा I: एक ट्रे न एक खम्भे को 8.8 seconds में और 207metres लं बे प्ले टफॉमस को 18 seconds में पार कर सकती है। ट्रे न द्वारा 567 km
की दूरी तय करने में थलया गया समय ज्ञात करें यफद िह अपनी गवत को 40% से बिा देती है।

मात्रा II: एक नाि िारा विरुि में 160km और साि ही िारा अनुप्रिाह में समान दुरी को 9 hours में तय कर सकती है जबवक िही नाि िारा
विरुि में 240 km की यात्रा 7.5 hours में कर सकती है। शांत जल में 180 km की दूरी तय करने में नाि द्वारा थलया गया समय ज्ञात करें।

मात्रा III: 17.5 m/s की गवत से यात्रा करने िाली कार द्वारा 315 km की दूरी तय करने में थलया गया समय ज्ञात करें।

(a) A, A (b) A, B (c) A, C (d) B, A (e) C, A

Any Error or Doubt Found Please Mail us to :-


WhatsApp @ 8445784192
10
Quant Updater – 20 | 24 June
Daily Quant Free Magazine for Bank Exam Students

16. In a set of even and composite numbers, the composite numbers are twice the number of even numbers and
the average of all the numbers of the set is 18. If the number of even numbers and composite numbers are
exchanged then the average of the set of numbers is increased by 4 and during the exchange of the numbers
the average of the even numbers and composite numbers individually remained constant, then find the ratio of
the average of composite numbers to the average of even numbers (initially).
एक सम और थमशित संख्याओ ं के समूह में, थमशित संख्याएूँ सम संख्याओ ं की संख्या से दोगुनी हैं और समुच्चय की सभी संख्याओ ं का औसत
18 है. यफद सम संख्याओ ं और थमशित संख्याओ ं को आपस में प्रवतिावपत वकया जाता है तो संख्याओ ं के समुच्चय का औसत 4 से बि जाता है
और संख्याओ ं के प्रवतिापन के दौरान सम संख्याओ ं और थमशित संख्याओ ं का औसत व्यधिगत रूप से स्थिर रहता है. तो थमशित संख्याओ ं
के औसत का सम संख्याओ ं के औसत से अनुपात ज्ञात करें (प्रारंथभक संख्या)
(a) 7 : 13 (b) 13 : 7 (c) 9 : 11
(d) 13 : 15 (e) Can’t be determined

17. Weights of Pawan, Balram, and Chinmay are in geometric progression while Weights of Pawan, Saquib and
Ramesh is in arithmetic progression. If ratio between common difference of arithmetic progression formed by
second group (Pawan, Saquib and Ramesh) and common ratio of geometric progression formed by first group
(Pawan, Balram, and Chinmay) is 2: 1 and total sum of Weights of first group (Pawan, Balram, and Chinmay) is 182
and total sum of second group (Pawan, Saquib and Ramesh) is 60 then which of the following options given below
will be the Weights of Pawan, Balram, and Chinmay respectively.
पिन, बलराम, और धचन्मय की आयु गुणोत्तर िेिी में हैं जबवक पिन, सावकब और रमेश की आयु समांतर िेणी में है. यफद दूसरे समूह (पिन,
सावकब और रमेश) द्वारा गरित समांतर िेणी के सामान्य अंतर और पहले समूह (पिन, बलराम, और धचन्मय) द्वारा गरित गुणोत्तर िेिी का
के सामान्य अनुपात का अनुपात 2: 1 है और पहले समूह (पिन, बलराम, और धचन्मय) की आयु का कुल योग 182 है और दूसरे समूह (पिन,
सावकब और रमेश) का कुल योग 60 है फफर नीचे फदए गए विकल्पों में से कौन सी क्रमशः पिन, बलराम, और धचन्मय की आयु होगी.
(a) 126, 42, 14 (b) 42, 14, 126 (c) 14, 42, 126
(d) 36, 60, 86 (e) Can’t be determined

Any Error or Doubt Found Please Mail us to :-


WhatsApp @ 8445784192
11
Quant Updater – 20 | 24 June
Daily Quant Free Magazine for Bank Exam Students

1. Solution (c) 28, 32, 41, 66, 115, ?


55 100
= × 150 + √28 × 7 − % × 999 The above series follows the pattern below,
100 9
= 82.5+ 14 -111 28 + (2)2 = 32
= -14.5 32 + (3)2 = 41
From I, -15 < -14.5 < -13 (True) 41 + (5)2 = 66
From II, -14.5 is a fraction number but < -14 66 + (7)2 = 115
From III, -14.5 is divisible by 5. So, multiple of 5. On comparing the the quantities,
Hence, statements I and III are true. Quantity 1 > Quanitity 2 > Quantity 3.
4. Solution (a)
2. Solution (d) Quantity 1,
=
150
× 20 +
323
+ √? = 9 × 9 5, 10, 40, 80, (?)
100 17
= 30 + 18 + √? =81 5 × 2 = 10

=48 + √? = 81 10 × 4 = 40

√𝑥 = 33 40 × 2 = 80

x = 1089 80 × 4 = 320

From I,
1089
= 121 and
1089
= 99 (True) The missing number in the series is 320.
9 11
Quantity 2,
From II, There are two natural numbers
15, 31, 63, 127, 255, 510.
between 31 and 34 i.e. 32 and 33
15 × 2 + 1 = 31
322 = 1024, 332 = 1089 (True)
1089 31 × 2 + 1 = 63
From III, = 11 (True)
99
63 × 2 + 1 = 127
127 × 2 + 1 = 255
3. Solution (b)
255 × 2 + 1 = 511
Quantity 1:
The wrong number in the series is 510.
1, 3, 5, 31, ?
Quantity 3,
The above series follows the pattern below,
342, 353, 341, 354, (?)
1×1+2=3
342 + 11 = 353
3×3–4=5
353 – 12 = 341
5 × 5 + 6 = 31
341 + 13 = 354
31 × 7 – 8 = 209
354 – 14 = 340
Quantity 2:
The missing number in the series is 340.
11, 15, 51, 132, 215.
Quantity 1 < Quantity 2 > Quantity 3.
The above series follows the pattern below,
11 + (2)2 = 15
5. Solution (c)
15 + (4)2 = 31 1rd
Prateek and Suleman complete , Chandan
31 + (6)2 = 67 3
1 1
completed × 2 =
67 + (8)2 = 131 12 6
1 1
131 + (10)2 = 231 Completed work before Vikas joined is +
3 6
1
∴ The wrong term in the series is 132. = , so to be completed by Vikas
2

Quantity 3:

Any Error or Doubt Found Please Mail us to :-


WhatsApp @ 8445784192
12
Quant Updater – 20 | 24 June
Daily Quant Free Magazine for Bank Exam Students
1 1 1 1
= 1 – = of work completed of work. is uncompleted
2 2 12 12
Now assignment uncompleted after Vikas work. Let x is no. of days in which Vikas can
1
left = complete whole work. So
3

So Vikas did = – = of work


1 1 1
Prateek’s work + Ankit’s work + Vikas’s work
2 3 6
He worked for 3 days, means he complete = 1 – uncompleted work
1 5 5 1
1/6th of work in 3 days. so he can do complete + + = 1–
6
12 8 𝑥 12
assignment A in × 3 = 18 days Solve, x = 24 = no. of days in which Vikas
1
6. Solution (d) can alone complete assignment 5.
Number of days in which Suleman and Prateek completed
1 th
work in 2 days, so he
12
Chandan can complete assignment 2 can complete whole assignment in
alone is 5x and 8x respectively. 2 × 12 = 24 days
Now if Vikas also joined for 4 days, Prateek = 24, Suleman = 20, Chandan = 30,
assignment would have been completed, Vikas = 24
means So together they can complete in –
1
+
1
+
6 3 4 4 24 20
+ + + =1 1 1 1
20 5𝑥 8𝑥 12 + = — 6 days
30 24 6
11 11
= 8. Solution (c)
10𝑥 30
x=3 2019:
So Suleman can complete assignment in The total goals scored in 2019 were 120.
5x = 15 days, and Chandan in 8x = 24 days. The average goals scored against Argentina
So
13
work, Suleman and Chandan together and England was 30 so the total goals scored
30

–(
1
+
1
)×𝑦 =
13 against Argentina and England was 60.
15 24 30
The total goals scored against others
Solve, y = 4 days
= 120 – 60 = 60
2020:
7. Solution (b)
The ratio of the total goals scored against
Suleman and Chandan together can
Others in 2019 to that of the total goals
complete work in 12 days
1 1 5 scored against Others in 2020 is 4 : 3.
[ + = — 12 days]
20 30 60 So the total goals against Others in 2020
Efficiency Ankit : (Suleman+Chandan) = 3/2 =
60
× 3 = 45
4
:1=3:2
The total goals scored against Argentina and
So, ratio of number days = 2 : 3
Others = 120
3 == 12 The total goals scored against Argentina
1 == 4 = 120 – 45 = 75
So Ankit can complete whole work in 2 == 8
2021:
days
The total goals scored in 2021 were 140.
Now Ankit worked for (2+3) = 5 days – total The sum of the total goals scored against
days for which Suleman and Chandan Argentina and England is equal to the total
worked.
goals scored against Others. It means the
5
So Ankit completed of work, Prateek
8

Any Error or Doubt Found Please Mail us to :-


WhatsApp @ 8445784192
13
Quant Updater – 20 | 24 June
Daily Quant Free Magazine for Bank Exam Students

total goals scored against Others were is half The total goals scored against England in
i.e 70 goals and the sum of the total goals 2021 = 40
scored against Argentina and England was So, required ratio = 45 : 40 = 9 : 8
70.
Years Argentina England Others 10. Solution (a)
2019 (120) 60 The total goals scored against Others in 2019
2020 75 45 = 60
2021 (140) 70 The total goals scored against Others in 2020
2019:
= 45
1rd
The goals scored against England were of
3 So, required difference = 60 – 45 = 15
the goals against Others in 2020.
So the total goals scored against England 11. Solution (d)
were Let the amount with Mahesh = 100x
45
= = 15 Amount invested in mutual funds = 20% of
3
The total goals scored against Argentina 100x = 20x
= 60 – 15 = 45 Amount spent on shopping = 15% of 100x = 15x
2021: Amount spent of food and rent = 25% of 100x
The total goals scored against Argentina = 25x
were twice of the goals scored against Remaining amount
England in 2019. = 100x - (20x + 15x + 25x) = 40x
The total goals scored against Argentina Amount invested in scheme A
= 15 × 2= 30 2
= 41 ( ) % 𝑜𝑓 40𝑥 = (
50𝑥
)
3 3
The total goals scored against England Amount invested in scheme B
= 70 – 30=40 50𝑥 70𝑥
= 40𝑥 − ( )= ( )
3 3
2020:
Amount of interest from scheme A
The total goals scored against England in 50𝑥 50𝑥 22𝑥
=( ) × [(1.2)2 − 1] = ( ) × 0.44 = ( )
2020 were equal to the total goals scored 3 3 3

Amount of interest from scheme B


against England in 2021. 70𝑥
[( 3 )× 20 × 2] 28𝑥
= = ( )
Years Argentina England Others 100 3

Total interest received


2019 (120) 45 15 60
22𝑥 28𝑥 50𝑥
=( )+ ( )= ( )
3 3 3

According to question-
2020 (160) 75 40 45 50𝑥
( ) - 15x = 2500
3
2021 (140) 30 40 70 5𝑥
( ) = 2500
3

The total goals scored in 2020 were 160. x = 1500


Amount of interest from scheme A
9. Solution (d) =(
22𝑥
) = Rs.11000
3
The total goals scored against Argentina in Amount of interest from scheme B
2019 = 45 =(
28𝑥
) = Rs.14000
3

Any Error or Doubt Found Please Mail us to :-


WhatsApp @ 8445784192
14
Quant Updater – 20 | 24 June
Daily Quant Free Magazine for Bank Exam Students

Let number of chairs and tables purchased Amount of interest from scheme A
by him is 'a' and '16 - a' respectively. =(
50𝑥
) × [(1.2)2 − 1] = (
50𝑥
) × 0.44 = (
22𝑥
)
3 3 3
Probability of selecting 2 chairs at random Amount of interest from scheme B
11
= C2/ C2 =
a 16
[(
70𝑥
)× 20 × 2] 28𝑥
20 = 3
= ( )
𝑎(𝑎 − 1) 100 3
[ ] 11
2 Total interest received
=
120 20 22𝑥 28𝑥 50𝑥
=( )+ ( )= ( )
a(a - 1) = 11 × 12 = 12 × (12 - 1) 3 3 3

According to question-
a = 12
50𝑥
( ) - 15x = 2500
Number of chairs purchased by him = a = 12 3
5𝑥
Number of tables purchased by him ( ) = 2500
3

= (16 - a) = 4 x = 1500
From the interest received from scheme A Amount of interest from scheme A
2 22𝑥
he purchased another item X at 16 ( )% =( ) = Rs.11000
3 3

discount. Amount of interest from scheme B


28𝑥
So, selling price of item X = Rs.11000 =( ) = Rs.14000
3
Marked price of item X Let number of chairs and tables purchased
100
= 11000 × ( ) = Rs.13200 by him is 'a' and '16 - a' respectively.
83.33

Number of chairs purchased by him = a = 12 Probability of selecting 2 chairs at random


11
Number of tables purchased by him = 4 = aC2/16C2 =
20
Probability that both the items are tables [
𝑎(𝑎 − 1)
] 11
1 2
= 4C2/16C2 = =
20 120 20
Probability that one is chair and another is a(a - 1) = 11 × 12 = 12 × (12 - 1)
2
table = ( 4C1 × 12C1)/16C2 = a = 12
5

Required probability = ( ) + ( ) =
1 2 9 Number of chairs purchased by him
20 5 20
= a = 12

12. Solution (a) Number of tables purchased by him

Let the amount with Mahesh = 100x = (16 - a) = 4

Amount invested in mutual funds = 20% of From the interest received from scheme A
2
100x = 20x he purchased another item X at 16 ( )%
3

Amount spent on shopping = 15% of 100x = 15x discount.


Amount spent of food and rent = 25% of 100x So, selling price of item X = Rs.11000
100
= 25x Marked price of item X = 11000 × ( )
83.33
Remaining amount = 100x - (20x + 15x + 25x) = Rs.13200
= 40x Selling price of item X when sold to Mahesh
2
Amount invested in scheme A = 41 ( )% of = Rs.11000
3

40x = (
50𝑥
) Marked price of item X = Rs.13200
3
100
Amount invested in scheme B Cost price of item X = 13200 × ( ) = Rs.10000
132

= 40x - (
50𝑥
)= (
70𝑥
) Selling price of item X when sold to Suresh
3 3

Any Error or Doubt Found Please Mail us to :-


WhatsApp @ 8445784192
15
Quant Updater – 20 | 24 June
Daily Quant Free Magazine for Bank Exam Students

= 115% of 10000 = Rs.11500 = a = 12


Amount of Discount offered = 13200 - 11500 Number of tables purchased by him
= Rs.1700 = (16 - a) = 4
Per cent of discount offered From the interest received from scheme A
1700 29
=( ) × 100 = 12 ( ) % he purchased another item X at 16 ( ) %
2
13200 33 3
discount.
13. Solution (b) So, selling price of item X = Rs.11000
Let the amount with Mahesh = 100x 100
Marked price of item X = 11000 × ( )
Amount invested in mutual funds = 20% of 83.33
= Rs.13200
100x = 20x
Invested capital of Mahesh = Rs.11000
Amount spent on shopping = 15% of 100x = 15x
Amount spent of food and rent = 25% of 100x Invested capital of Sunil = Rs.9000
= 25x Ratio of their profit = (11000 × 4 + 4000 × 3):
Remaining amount = 100x - (20x + 15x + 25x) (9000 × 7) = 56: 63 = 8: 9
= 40x Profit amount received from Sunil
9
Amount invested in scheme A = 41 ( ) % of
2
= 21250 × ( ) = Rs.11250
3 17
50𝑥
40x = ( )
3
Amount invested in scheme B 14. Solution (d)
= 40x - (50𝑥/3) = (
70𝑥
) Quantity I:
3
Amount of interest from scheme A Total number of outcomes when two dices
=(
50𝑥
) × [(1.2)2 − 1] = (
50𝑥
) × 0.44 = (
22𝑥
) are rolled = 6 × 6 = 36
3 3 3
Amount of interest from scheme B Favourable outcomes = {(2, 6), (3, 5), (4, 4), (5,
[(
70𝑥
)× 20 × 2] 28𝑥
3), (6, 2)}
= 3
= ( )
100 3 Number of favourable outcomes = 5
Total interest received 5
22𝑥 28𝑥 50𝑥
Required probability =
36
=( )+ ( )= ( ) 5 10
3 3 3 So, Quantity I = =
According to question- 36 72

50𝑥 Quantity II:


( ) - 15x = 2500
3
5𝑥
Total number of outcomes = 23 = 8
( ) = 2500
3 Sample space = {HHH, HHT, HTT, HTH, TTH, THH,
x = 1500 THT, TTT}
Amount of interest from scheme A
Number of times we are getting at most 2
22𝑥
=( ) = Rs.11000 heads = 7
3
Amount of interest from scheme B Required probability =
7
28𝑥 8
=( ) = Rs.14000 7 63
3 So, Quantity II = =
8 72
Let number of chairs and tables purchased
Quantity III:
by him is 'a' and '16 - a' respectively.
Total number of outcomes = 6 × 6 = 36
Probability of selecting 2 chairs at random
11 Favourable outcomes = {(1, 1), (2, 2), (3, 3), (4, 4),
= aC2/16C2 =
20 (5, 5), (6, 6)}
𝑎(𝑎 − 1)
[ ] 11 Number of favourable outcomes = 6
2
= 6 1
120 20 Required probability = =
a(a - 1) = 11 × 12 = 12 × (12 - 1)
36 6
1 12
So, Quantity III = =
a = 12 6 72

Number of chairs purchased by him Therefore, Quantity I < Quantity II > Quantity III

Any Error or Doubt Found Please Mail us to :-


WhatsApp @ 8445784192
16
Quant Updater – 20 | 24 June
Daily Quant Free Magazine for Bank Exam Students
2Px+Cx
15. Solution (a) = 22 ⇒ 2P + C = 66 …(ii)
3x
Quantity I: Solving eqn. (i) and (ii), we get
Let speed of the train is ‘x’ m/s and length of 2P + C = 66
( P + 2C = 54) × 2
train is ‘l’ metres. – – –
So, l = 8.8 × x = 8.8x – 3C = – 42
And, l + 207 = 18 × x ⇒ C = 14
66 –14
Or, 8.8x + 207 = 18x ∴P= = 26
2
14 7
Or, 9.2x = 207 ∴ Required ratio = =
26 13
18
Or, x = 22.5 m/s = 22.5 × ( ) = 81 km/h
5
17. Solution .(c)
Increased speed = 1.40 × 81 = 113.4 km/h
567 Let Weights of Pawan, Balram, and Chinmay
Desired time = = 5 hours
113.4 are a, ar and ar² respectively.
So, Quantity I = 5 hours (∵ Pawan, Balram, and Chinmay → G.P.)
Quantity II: Weights of Pawan, Saquib and Ramesh are a,
240
Speed of boat in upstream = = 32 km/h a+ d, a +2d
7.5
Let speed of boat in downstream be ‘x’ km/h (∵ Pawan, Saquib and Ramesh → A.P.)
So, (160/32) + (
160
)=9 Where d= common difference
𝑥
160 r = common ratio
Or, ( )=9–5=4
𝑥 According first condition
Or, x = 40 d 2
= ⇒ d = 2r …(i)
r 1
Speed of boat in still water
32 + 40
According to second condition
={ } = 36 km/h a(r 3 − 1) 3
2
180 = 182 & [2a + 2d] = 60
Desired time = = 5 hours (r − 1) 2
36
⇒ a (r² + r + 1) = 182 & (a + d) = 20
So, Quantity II = 5 hours
From these equations, we can find a new
Quantity III:
18 equation
Speed of car = 17.5 × ( ) = 63 km/h
5 (20 – 2r) (r² + r + 1) = 182
315
Desired time = ( ) = 5 hours (∵ d = 2r & a = 20 –d)
63
So, Quantity III = 5 hours ⇒ (10– r) (r²+ r+ 1) = 91
Therefore, Quantity I = Quantity II = Quantity III ⇒ r³ – 9r² – 9r + 81 = 0
⇒ r² (r – 9) – 9 (r – 9) = 0
16. Solution (a) ⇒ (r – 9) (r² – 9) = 0
Let average of even numbers = P ⇒ r = 9, 3, –3
Average of composite numbers = C a = 20 – 2 × 9 or 20 – 2 × 3
Let no. of even numbers = x = 2 or 14
∴ no. of composite numbers = 2x ∴ Weights of Pawan, Balram, and Chinmay →
According to first condition, 2, 18, 162, or 14, 42, 126
Px+2Cx
= 18 From options the correct ans is 14, 42 and 126
3x
⇒ P + 2C = 54 …(i) kg
According to second condition,

For All Banking & Insurance Exam

PW Web/App - https://smart.link/7wwosivoicgd4

Library- https://smart.link/sdfez8ejd80if

Any Error or Doubt Found Please Mail us to :-


WhatsApp @ 8445784192
17

You might also like